LSAT and Law School Admissions Forum

Get expert LSAT preparation and law school admissions advice from PowerScore Test Preparation.

 Administrator
PowerScore Staff
  • PowerScore Staff
  • Posts: 8917
  • Joined: Feb 02, 2011
|
#63966
Complete Question Explanation

Resolve the ParadoxX. The correct answer choice is (E)

The paradox presented here is fairly simple: In Springfield the average worker lives farther from
work than in Rorchester, so we might expect Springfield to have a greater demand for public
transportation. Yet Springfield has half as many bus routes as Rorchester.

This stimulus is followed by a ResolveX question stem. Thus, the four incorrect answer choices will
help to resolve the apparent discrepancy discussed in the stimulus, and the correct answer we seek
will fail to provide a resolution.

Answer choice (A): If the vast majority of Springfield’s workforce is employed outside the city, then
this would help to explain the lower number of bus routes necessary within the city—three quarters
of the working population leaves the city on a daily basis.
Since this answer choice provides a reasonable resolution to the paradox presented in the stimulus, it
cannot be the correct answer choice to this Resolve Except question.

Answer choice (B): Like incorrect answer choice (A) above, this choice also provides resolution to
the apparent discrepancy: a greater average number of cars would make for less need for bus routes.
Since this choice resolves the paradox, it must be one of the four incorrect answer choices here.

Answer choice (C): If Rorchester has fewer railway lines, and Springfield has a relatively greater
number, this fact would help to explain why fewer bus routes would be necessary in Springfield,
helping to provide resolution to the paradox presented in the stimulus. Since this choice does help
resolve, it cannot be the correct answer choice.

Answer choice (D): Springfield’s relatively smaller number of routes would be explained in part by
routes of greater lengths and frequency. This answer choice helps to resolve the paradox, so it must
be one of the four incorrect answer choices in this case.

Answer choice (E): This is the correct answer choice, as it is the only choice which fails to
resolve the paradox presented in the stimulus. This answer actually expands the paradox, because
a greater population in Springfield would give us even more reason to expect a higher demand for
public transportation there, making Springfield’s relatively low supply of busses that much greater a
paradox.
 emilysnoddon
  • Posts: 64
  • Joined: Apr 22, 2016
|
#23718
I was debating between answer C and answer E. I am confused about why E is correct and C is not.
User avatar
 Stephanie Oswalt
PowerScore Staff
  • PowerScore Staff
  • Posts: 811
  • Joined: Jan 11, 2016
|
#23783
Dear Emily,

Thanks for your question. Generally speaking, we need a bit more input from you before we delve into a discussion of a particular LR question. Ultimately, it won't be us who are taking the test; it's you! :-) Our goal is to help you cultivate the analytical ability to approach these questions on your own, which is why you need to help us help you first.

Here's what I'd like you to do:
  • 1. Describe your approach to the stimulus. Did you understand the argument, if any, from a structural standpoint? What is the conclusion, and what evidence is the author using in support of that conclusion?

    2. Did you prephrase an answer to the question in the stem? If so, what was your prephrase?

    3. What exactly made the two answer choices you have listed particularly attractive? Did you use any question type-specific test (e.g. Assumption Negation Technique) to differentiate between them?
Thanks,

Stephanie
 htngo12
  • Posts: 40
  • Joined: May 19, 2016
|
#27248
Hi!

The argument presents on average Springfield live farther from their workplaces than Rochester residents.
With Springfield having only 1/2 the bus routes. Therefore, one would expect the demand for public transportation to be greater in Springfield.

Since this is Resolve the Paradox (X) question, I am looking for an answer that does not resolve either side.

I originally choose (E) Springfield has a larger population than Rochester does, but looking back at the argument and inputing this answer into it would explain the expected demand for public transportation.

Since the correct answer (C) Rochester has fewer railways lines then Springfield could be read as Springfield has more railways then Rochester. Then one would not expect the demand for public transportation to be greater in Springfield.

Would this reasoning be an appropriate way to look at the argument?
 Clay Cooper
PowerScore Staff
  • PowerScore Staff
  • Posts: 241
  • Joined: Jul 03, 2015
|
#27261
Hi htngo,

I think maybe you have gotten mixed up - answer choice E is correct, because it does nothing to resolve the paradox (if Springfield's population was bigger than Rorchester, that just makes it even more inexplicable that Springfield can get by with fewer bus routes); answer choice C, however, does in fact resolve the paradox and is therefore incorrect (if Rorchester has fewer railway lines than Springfield, that might explain why it needs more buses).

It looks from your post like you got mixed up and thought C was correct and E was incorrect. If that happened, don't be discouraged, I've done it roughly eight million times.

Keep working hard!
 htngo12
  • Posts: 40
  • Joined: May 19, 2016
|
#27338
Yes, I did get it mixed up:( I guess it's a by-product of going through so many questions.

So (E) the correct answer does not resolve the paradox because it mentions Springfield having a larger population does not mean that all of the larger population would rely on public transportation. They could carpool, they could not work etc..

With (C) mentions about Rochester having less railways then would resolve the paradox.

Got It!

Get the most out of your LSAT Prep Plus subscription.

Analyze and track your performance with our Testing and Analytics Package.